Can Calculating Cumulative Binomial Probabilities Be Simplified?

Click For Summary
The discussion centers on calculating the probability of fewer than four base metal failures in a sample of 20 weld failures, where 15% are base metal failures. A user inquires if there is a quicker method than summing individual probabilities for 0 to 3 failures. The response suggests that while the direct method is straightforward, a recursive approach can expedite the calculations. By using the relationship between successive probabilities, one can derive the probabilities more efficiently. This method allows for a streamlined calculation without losing accuracy.
Mdhiggenz
Messages
324
Reaction score
1

Homework Statement



Of all the weld failures in a certain assembly, 85%
of them occur in the weld metal itself, and the remaining
15% occur in the base metal. A sample of
20 weld failures is examined.

a. What is the probability that fewer than four of
them are base metal failures?

Is there a faster way to solve rather than doing p(x=0)+p(x=1)+p(x=2)+p(x=3)?

Thanks

Brandon


Homework Equations





The Attempt at a Solution

 
Physics news on Phys.org
Mdhiggenz said:

Homework Statement



Of all the weld failures in a certain assembly, 85%
of them occur in the weld metal itself, and the remaining
15% occur in the base metal. A sample of
20 weld failures is examined.

a. What is the probability that fewer than four of
them are base metal failures?

Is there a faster way to solve rather than doing p(x=0)+p(x=1)+p(x=2)+p(x=3)?

Thanks

Brandon


Homework Equations





The Attempt at a Solution


No, the way you said is about as short as possible. You can sometines speed things up a bit by doing it recursively: if
P(k) = {n \choose k} p^k (1-p)^{n-k}
we have
\frac{P(k+1)}{P(k)} = r(k) \equiv \frac{n-k}{k+1} \frac{p}{1-p},
so if we start from ##P(0) = (1-p)^n##, we can get ##P(1) = r(0) P(0),## ##P(2) = r(1) P(1),## etc.
 
Question: A clock's minute hand has length 4 and its hour hand has length 3. What is the distance between the tips at the moment when it is increasing most rapidly?(Putnam Exam Question) Answer: Making assumption that both the hands moves at constant angular velocities, the answer is ## \sqrt{7} .## But don't you think this assumption is somewhat doubtful and wrong?

Similar threads

Replies
4
Views
2K
  • · Replies 8 ·
Replies
8
Views
2K
  • · Replies 13 ·
Replies
13
Views
2K
  • · Replies 5 ·
Replies
5
Views
2K
  • · Replies 2 ·
Replies
2
Views
2K
  • · Replies 7 ·
Replies
7
Views
2K
  • · Replies 3 ·
Replies
3
Views
2K
  • · Replies 3 ·
Replies
3
Views
3K
  • · Replies 15 ·
Replies
15
Views
2K
Replies
2
Views
1K